BRS Pathology 20 - Endocrine System, BRS Pathology 10 - The Heart, BRS Pathology ALL QUESTIONS, BRS Pathology - Endocrine, Diseases of the Heart, The Endocrine System and Homeostasis, Endocrine (w/nutrition, breast and MSK pathology)- BRS/Robbins

Ace your homework & exams now with Quizwiz!

The answer is E. The history is that of pulmonary embolism and infarction, a danger of immobilization, and the postoperative state. The infarct consists of an area of coagulative necrosis with superimposed hemorrhage, a combination referred to as a hemorrhagic, or red, infarct. Red infarcts are typical of tissues with a redundant arterial blood supply. Prominent examples are the lung with its double circulation from the pulmonary and bronchial arteries and the gastrointestinal tract with its multiple anastomoses between branches of the mesenteric artery. When a portion of the blood supply is obstructed, other portions remain patent, which can lead to hemorrhage into the infarcted area.

Two days following a cholecystectomy, a 32-year-old hospitalized woman has sudden onset of dyspnea, pleural pain, and cough productive of frothy, blood-tinged sputum. Ventilation-perfusion scintigraphy indicates a perfusion defect. If it were possible to examine a portion of the affected lung, which of the following would most likely have been found? (A) Air embolism (B) Anemic (white or pale) infarct (C) Disseminated intravascular coagulation (DIC) (D) Generalized thrombosis (e) Hemorrhagic (red) infarct

The answer is A. The clinical presentation is that of the cri du chat (or 5p-) syndrome.

A 2-year-old child has been followed for mental retardation and slow development, as well as multiple birth defects. The child has a high-pitched catlike cry. On examination, microcephaly, hypertelorism, micrognathia, epicanthal folds, low-set ears, and hypotonia are noted. Karyotypic analysis would be expected to show (A) 5p-. (B) 22q11-. (C) 45,XO. (D) 46,XY. (E) 47,XXY.

The answer is A. Yellowing of the sclerae, skin, and oral mucosa are all characteristic of jaundice, the accumulation of bilirubin, the catabolic product of the heme moiety of hemoglobin. Jaundice can occur by diverse mechanisms: hemolytic (see Chapter 11), hepatocellular (see Chapter 16), or obstructive (see Chapter 16).

A 20-year-old man presents with yellowing of the sclerae, skin, and oral mucosa. Which of the following accumulations underlies these findings? (A) Bilirubin (B) Hemosiderin (C) Lead (D) Melanin (E) Silver

The answer is A. The diagnosis is X-linked agammaglobulinemia of Bruton. Failure of maturation of pre-B cells is associated with absence of mature B lymphocytes and plasma cells; failure of antibody synthesis; marked serum hypogammaglobulinemia; and recurrent bacterial infections, especially sinopulmonary infections. Histologic examination of lymphoid tissue reveals marked underdevelopment of germinal centers. T cells are unaffected, as are T-cell functions such as cell-mediated immunity and resistance to most viral infections. The disease is X-linked and is due to mutations in the B cell tyrosine kinase (Btk) gene.

A 1-year-old boy has had repeated pyogenic infections with streptococci, staphylococci, and Haemophilusfor the past 6 months. Tests for T-cell function, granulocyte function, and complement activity have all been unaffected. Serum IgG is 50 mg/dL (normal 500 mg/dL). Flow cytometry revealed absent expression of heavy-chain µon blood lymphocytes. T lymphocytes were slightly increased in number, with a normal CD4+to CD8+ ratio. Expected findings on examination of a lymph node biopsy from this patient include which of the following? (A) Absent germinal centers (B) Follicular hyperplasia with exuberant proliferation of immature B cells (C) Massive TH1 cell infiltration into lymphoid follicles (D) Normal lymphoid tissue development (E) Plasma cell hyperplasia

The answer is A. The diagnosis is cystic fibrosis, the most common lethal genetic disease in Caucasian populations. The disorder is due to a defect in the cystic fibrosis transmembrane conductance regulator protein, and about 70% of cases have a deletion of phenylalanine in position 508 (DF508 mutation). Affected patients often have multiple pulmonary infections and pancreatic insufficiency with steatorrhea and failure to thrive. Death is often due to respiratory failure secondary to repeated pulmonary infections, facilitated by the buildup of thick, tenacious mucus in the airways. Increased concentration of chloride in sweat and tears is characteristic, and the sweat test is an important diagnostic adjunct.

A 1-year-old female infant is hospitalized for pneumonia. Bacterial cultures of the sputum have grown Pseudomonas aeruginosa. She has had two prior hospitalizations for severe respiratory infections. Her mother has noted that when she kisses her child, the child tastes "salty." The child has had weight loss that the mother attributes to frequent vomiting and diarrhea with bulky, foul-smelling fatty stools. The child is small for her age. Which of the following critical proteins is altered in this condition? (A) Cystic fibrosis transmembrane conductance regulator (B) Dystrophin (C) α-1,4-Glucosidase (D) α-L-Iduronidase (E) Lysyl hydroxylase

The answer is B. Acute cellular rejection is characterized by an infiltrate of both CD4+ and CD8+ lymphocytes. Acute rejection occurs over a variable time period, ordinarily days to weeks to months after transplant.

A 25-year-old woman with membranous glomerulonephritis receives a kidney transplant. The donor is her HLA-matched sister. She does well initially, but after several weeks, there is a progressive increase in serum creatinine. Assuming that this represents acute cellular rejection, an infiltrate with which of the following types of inflammatory cells is most likely to be a prominent finding on renal biopsy? (A) Eosinophils (B) Lymphocytes (C) Mast cells (D) Monocytes-macrophages (E) Neutrophils

The answer is E. Rejection of the stem cell transplant is occurring, as evidenced by reappearance of the marker for the patient's original gene. Small nucleotide polymorphisms are the most frequent form of DNA variation. As the name implies, they are typically small in size, often a single nucleotide. They may occur in any portion of the gene, even in intergenic regions of the genome, and are of increasing importance as genetic markers, as illustrated by the example presented here.

A 1-year-old girl with an inborn error of metabolism resulting in a lysosomal storage disease receives a hematopoietic stem cell transplant intended to replace her macrophage population. The gene of interest has a "marker" small nucleotide polymorphism within a noncoding intron of the affected gene in which an A (patient gene) is substituted for a G (donor gene). She does quite well for the first 3 weeks. She tests positive for the missing enzyme, her previously abnormally enlarged organs begin to diminish in size, and assay of peripheral blood lymphocytes reveals increasing numbers of cells with the G polymorphism. However, the attending physicians are now concerned because repeated genetic testing reveals a progressive increase in lymphocytes with the A polymorphism. Which of the following is the best explanation for this finding? (A) Generalized immune complex formation (B) Graft-versus-host disease (C) Immune paralysis (D) Immune tolerance (E) Rejection of the stem cell transplant

The answer is D. The history is that of a craniopharyngioma, a benign tumor that does not invade or metastasize; however, local effects of this tumor can be quite destructive, and recurrence due to incomplete resection is not uncommon. Local growth and tissue destruction result in both anterior and posterior pituitary dysfunction, and a patient often presents with signs of increased intracranial pressure, sometimes with hydrocephalus and frequently with bilateral hemianopsia (loss of peripheral visual fields) due to impingement on the optic chiasm. Diabetes insipidus is also frequent. Calcification apparent on radiograph is often prominent, facilitating diagnosis.

A 10-year-old boy presents with headache and bilateral hemianopsia, as well as evidence of increased intracranial pressure and diabetes insipidus. Suprasellar calcification is apparent on radiographic examination. Resection of the contents of the sella turcica and parasellar area yields a large tumor with histology closely resembling the enamel organ of the embryonic tooth. The most likely outcome of this lesion is (A) local invasion and intracranial metastasis. (B) hematogenous metastasis to distal sites. (C) lymphatic spread to distal sites. (D) possible local recurrence with continued pressure-related damage to adjacent structures.

The answer is E. This is a case of acute rheumatic fever. Acute rheumatic fever manifests most commonly in patients 5 to 15 years of age with migratory polyarthritis, pancarditis, subcutaneous nodules, erythema marginatum, and Sydenham chorea. Decades later, severe valvular disease, often manifesting as mitral stenosis, may develop as a feature of rheumatic heart disease. In this chronic stage of rheumatic disease, fibrotic valves may become stenotic, insufficient, or both, but much more commonly, progression to cardiac valve complication does not occur.

A 10-year-old boy presents with migratory polyarthritis involving several large joints, fever, and malaise. Physical examination reveals a new heart murmur and friction rub on auscultation, and a painless nodule is detected on the extensor surface of the elbow. He had a severe sore throat approximately 2 weeks ago, apparently recovering without antibiotic therapy. The anti- streptolysin O (ASO) titer is elevated. Which of the following describes the most likely outcome for this patient? (A) Development of mitral valve stenosis over many months to years (B) Development of mitral valve stenosis over the next few months (C) Increasing severity of the current symptoms and findings over the next few decades (D) Persistence of the current symptoms and signs over the patient's lifetime (E) Total recovery after 1 to 2 months with no further complications or sequelae

The answer is D. The clinical findings are suggestive of Addison disease (primary adrenocortical insufficiency). About 70% of cases are now due to autoimmune adrenalitis, but until recently the most frequent cause was tuberculosis. Hyperpigmentation in Addison disease results from compensatory hypothalamic production of proopiomelanocortin, the precursor peptide of both corticotropin and melanocyte-stimulating factor.

A 14-year-old boy is seen because of increasing weakness, easy fatigability, and weight loss over the past 3 months. In addition, he has recently developed nausea, vomiting, and abdominal pain. His blood pressure is markedly decreased, and he has increased pigmentation of his skin creases. These findings are suggestive of (A) Cushing syndrome. (B) secondary hyperaldosteronism. (C) osteitis fibrosa cystica. (D) Addison disease. (E) 1α-Hydroxylase deficiency.

The answer is C. Although most patients with Turner syndrome have a 45,XO karyotype, the syndrome is thought to be caused by the absence of one set of genes from the short arm of one X chromosome, and a variety of chromosome abnormalities may be found. Many patients are mosaics (e.g., 45,XO/46,XX or 45,XO/47,XXX), and the phenotype is highly variable. A deletion of the SHOXgene can cause an identical phenotype and may be considered to be a variant of Turner syndrome.

A 14-year-old girl with amenorrhea is concerned because of the delayed onset of menses. She has shortened stature and a wide, webbed neck; broad chest; and secondary sexual characteristics consistent with those of a much younger girl. Which of the following chromosomal changes is most consistent with these findings? (A) 5p- (B) 22q11- (C) 45,XO (D) 46,XY (E) 47,XXY

The answer is C. Retcodes for a transmembrane receptor tyrosine kinase that is mutated in the MEN IIa and MEN IIb syndromes, as well as in sporadic cases of medullary carcinoma of the thyroid. Bcr-ablfusion results from the chromosomal translocation of chronic myelogenous leukemia. N-mycis amplified in neuroblastoma. Amylin derived from islet amyloid polypeptide accumulates in the pancreatic islets in type 2 diabetics.

A 15-year-old boy presents to the endocrinologist with multiple mucocutaneous neuromas and a marfanoid habitus (tall with long extremities). His older brother has had a thyroidectomy for medullary carcinoma of the thyroid and later has been diagnosed with bilateral tumors of the adrenal medulla. It is likely that further investigation in both brothers will demonstrate an abnormality in which of the following genes or gene products? (A) Bcr-abl (B) N-myc (C) Ret (D) Amylin (E) Insulin-associated polypeptide

The answer is E. Myelin figures, cell blebs, mitochondrial swelling, and glycogen depletion are all signs of reversible injury. Nuclear changes such as pyknosis, karyorrhexis, and karyolysis are signs of cell death and are, of course, irreversible.

A 56-year-old man dies 24 hours after the onset of substernal chest pain radiating down his left arm to the ulnar aspect of his fingertips. Which of the following morphologic myocardial findings is an indicator of irreversible injury? (A) Cell blebs (B) Depletion of glycogen (C) Mitochondrial swelling (D) Myelin figures (E) Pyknotic nuclei

The answer is E. The clinical findings are typical of acute appendicitis, another example of severe acute inflammation. Because the danger of perforation is great, early appendectomy is the treatment of choice. Suppurative or purulent inflammation is characterized by the prominent areas of edema resulting from increased vascular permeability, congestion, and a purulent (pus-containing) exudate consisting of necrotic cells and large numbers of neutrophils. In addition, other signs of acute inflammation, such as congestion, are prominent. The patient responds with the sensation of pain (induced by increased hydrostatic pressure in tissue and by chemical mediators such as bradykinin) and the acute phase reaction (in this instance, fever and neutrophilic leukocytosis with a "shift to the left").

A 16-year-old boy presents with a 24-hour history of severe abdominal pain, nausea, vomiting, and low-grade fever. The pain is initially periumbilical in location but has migrated to the right lower quadrant of the abdomen, with maximal tenderness elicited at a site one-third of the way between the crest of the ileum and the umbilicus (McBurney point). The leukocyte count is 14,000/mm3, with 74% segmented neutrophils and 12% bands. Surgery is performed. Which of the following describes the expected findings at the affected site? (A) Fistula (abnormal duct or passage) connecting to the abdominal wall (B) Granulation tissue (new vessels and young fibroblasts) with a prominent infiltrate of eosinophils (c) Granulomatous inflammation with prominent aggregates of epithelioid cells and multinucleated giant cells (d) Massive infiltration of lymphocytes and plasma cells (e) Prominent areas of edema, congestion, and a purulent reaction with localized areas of abscess formation

The answer is A. The patient has renal agenesis, absence of the kidney due to failure of organ development. The congenital lack of one kidney differs from atrophy, in which a decrease in the size of an organ results from a decrease in the mass of preexisting cells. Unilateral renal agenesis is usually a harmless malformation, and the opposite kidney is often enlarged due to compensatory hypertrophy. Bilateral renal agenesis is incompatible with life and is of special interest since it can lead to the Potter progression (see Chapter 17).

A 16-year-old girl undergoes radiologic imaging of her abdomen and is found to have only one kidney. She had been entirely unaware of this problem. Which of the following terms is most descriptive of this finding? (A) Agenesis (B) Atrophy (C) Hyperplasia (D) Hypoplasia (E) Metaplasia

The answer is D. Marfan syndrome, an autosomal dominant disorder caused by mutations of the fibrillin gene on chromosome 15, is a frequent cause of ectopia lentis. Other cardinal features include skeletal and cardiovascular abnormalities. Patients are tall and thin, with notably long limbs and digits. An anterior chest deformity known as pectus excavatum is sometimes seen, and vertebral abnormalities include scoliosis and lordosis. In addition, a highly arched palate and crowding of the teeth may occur. Cardiovascular complications include mitral valve prolapse and mitral regurgitation. Cystic medial necrosis can lead to dilation of the aortic root and aortic regurgitation. Life-threatening complications are aortic aneurysm and aortic dissection.

A 19-year-old college sophomore is referred by his ophthalmologist because of the finding of ectopia lentis (dislocation of the lens), which has resulted in visual difficulties that have interfered with his performance on the varsity basketball team. The patient is very tall, with long limbs and long, slender, spiderlike fingers. His chest has a "caved-in" appearance, and he also has a modest degree of scoliosis. A midsystolic "click" is heard, and an echocardiogram reveals mitral valve prolapse. The most likely diagnosis is (A) Ehlers-Danlos syndrome. (B) Fabry disease. (C) Hurler syndrome. (D) Marfan syndrome. (E) Pompe disease.

The answer is A. Either of two coreceptors, CCR5 or CXCR4, is involved in the initial binding of the virus to the cell surface molecule gp120 on CD4+cells. It is of considerable interest that certain mutations in the CCR5 gene are associated with what appears to be total resistance to infection with some common strains of HIV. Homozygotes are totally resistant, and heterozygotes develop a more slowly progressive disease.

A 19-year-old intravenous drug user has regularly sought human immunodeficiency virus (HIV) testing, always with negative results. He admits to carelessly sharing needles on multiple occasions with individuals later found to be HIV-positive. He has heard that there is an inherited genetic basis for some people to be relatively "immune" to HIV infection. The genetic change that he is referring to is a mutation in a gene coding for which of the following proteins? (A) CCR5 (B) CD4 (C) gp120 (D) gp41 (E) Reverse transcriptase

The answer is C. Wiskott-Aldrich syndrome is an X-linked disorder characterized by eczema, thrombocytopenia, recurrent infections, and poor antibody response to polysaccharide antigens. Bloody diarrhea is also common. Death before 6 years of age occurs frequently and is most often due to bleeding, infection, or malignancy (most often lymphoma).

A 2-year-old boy has eczema and thrombocytopenia. There is also a history of recurrent infection. His brother has similar abnormalities, but none of his three sisters is affected. Patients with this disorder are known to have impaired antibody response to which of the following types of antigen? (A) Deoxyribonucleic acid (B) Phospholipid (C) Polysaccharide (D) Ribonucleic acid (E) Steroid

The answer is B. Chronic granulomatous disease of childhood, a condition characterized by repeated infections and most commonly X-linked inheritance, is marked by failure of the myeloperoxidase-halide system of killing within phagocytic cells. It is caused by the deficiency of NADPH oxidase activity. This results in a secondary deficiency of reactive oxygen metabolites, including H2O2, which, along with halide ions, functions as a substrate for myeloperoxidase. A hallmark of the disorder is the failure of intracellular killing of catalase-positive organisms, exemplified by staphylococci. These organisms are ingested but not killed. The impaired phagocytic cell is incapable of producing H2O2, and any H2O2 produced by the microorganism itself is inactivated by endogenous catalase. In contrast, catalase-negative microorganisms, such as streptococci, are ingested and killed. They too produce endogenous H2O2, which is thus available as one of the substrates for myeloperoxidase. In a sense, the microorganisms assist in their own killing.

A 2-year-old boy presents with recurrent infections involving multiple organ systems. Extensive investigation results in a diagnosis of chronic granulomatous disease of childhood. Which of the following most closely characterizes the abnormality in this patient's phagocytic cells? (A) Decreased killing of microorganisms because of enhanced production of hydrogen peroxide (B) Deficiency of NADPH oxidase activity (c) Impaired chemotaxis and migration caused by abnormal microtubule formation (d) Inability to kill streptococci (e) Increased myeloperoxidase-halidemediated killing of catalase-positive organisms when compared with catalase-negative organisms

The answer is C. Theoretically, a person who carries a robertsonian translocation with chromosome 21 and a second acrocentric chromosome has a 1 in 3 chance of having a child with trisomy 21; however, the risk of a live birth of a child with Down syndrome is actually much less, presumably because of a high incidence of spontaneous abortion of such fetuses. The important point is that a robertsonian translocation predisposes to a hereditable form of Down syndrome. The risk is not related to maternal age and is much higher than the risk in the general population, which is 1 in 1500 for women 20 years of age and younger, increasing to 1 in 25 in women older than 45 years of age.

A 20-year-old woman has a robertsonian translocation involving chromosome 21 and a second acrocentric chromosome. What is the theoretic likelihood of a functional trisomy 21 if one of her ova is fertilized by a normal sperm? (A) 1 in 1 (B) 1 in 2 (C) 1 in 3 (D) 1 in 4 (E) 1 in 1500

The answer is C. The diagnosis is systemic lupus erythematosus, and the most characteristic lesions are mediated by immune complex deposition (type III hypersensitivity). In this form of hypersensitivity, antibody combines with antigen, resulting in antigen-antibody complexes. Insoluble aggregates of immune complex are deposited in vessel walls, serosal surfaces, and other extravascular sites, and complement is bound. The antigen-antibody-complement complexes are highly chemotactic for neutrophils, which release lysosomal enzymes and other mediators of tissue damage (prostaglandins, kinins, and free radicals).

A 20-year-old woman presents with malar rash, arthralgias, low-grade fever, and high titer antibodies to double-stranded DNA and to the Sm (Smith) antigen. Which of the following forms of hypersensitivity is the primary mechanism of the abnormalities found in this disorder? (A) Type I (immediate or anaphylactic) hypersensitivity (B) Type II (antibody-mediated or cytotoxic) hypersensitivity (C) Type III (immune complex-mediated disorders) hypersensitivity (D) Type IV (cell-mediated) hypersensitivity

The answer is B. Graft-versus-host disease is most often manifested by clinical findings related to the three principal target organs: the skin, liver, and gastrointestinal tract. The skin manifestations are often initiated by a pruritic rash. Elevation of bilirubin and liver enzymes signals the hepatic involvement. Effects on the ileum and colon present as diarrhea and abdominal pain. The lesions are caused by donor lymphocytes, with targeting of host epithelial cells by CD8+T cells.

A 22-year-old woman with acute myeloblastic leukemia receives an allogeneic bone marrow transplant with apparent successful engraftment. Three weeks later, early jaundice, as well as a generalized maculopapular rash, is noted. Profuse diarrhea follows. A skin biopsy reveals vacuolar changes, necrotic epidermal cells, and a lymphocytic infiltrate. These findings are most likely caused by (A) antibody-dependent cellular cytotoxicity. (B) attack on host epithelial cells by donor CD8+T cells. (C) contamination of the donor transplant cells with hepatitis C virus. (D) IgE and mast cell-mediated anaphylactic hypersensitivity. (E) secretion of IL-2 and interferon-γ by TH1 cells.

The answer is C. The patient has fat embolism syndrome, which is characterized by pulmonary distress, cutaneous petechiae, and various neurologic manifestations. Fat embolism is a well-known complication of fractures of long bones, such as the femur, and other bones with abundant fatty marrow. On fracture, marrow fat can enter the circulation, and small fat droplets can lodge in the vessels of the skin, lung, and microvasculature of the brain, resulting in the clinical manifestations of this disorder.

A 23-year-old man undergoes surgery for fractures of the pelvis and left femur resulting from a high-speed motor vehicle accident. The following day he develops dyspnea, speech difficulties, and a petechial skin rash. Which of the following types of embolism is the likely cause of these findings? (A) Air (B) Amniotic fluid (C) Fat (D) Paradoxical (e) Thrombotic

The answer is A. The patient presents with signs and symptoms of Graves disease, an autoimmune disorder in which patients develop autoantibodies that stimulate thyroid hormone production. As with many other autoimmune disorders, there is an association with certain human leukocyte antigens (HLAs); in the case of Graves disease, it is HLADR3 and HLA-B8.

A 23-year-old woman presents with tremor, restlessness, heat intolerance, palpitation, and unexplained weight loss. The thyroid is symmetrically enlarged, the pulse is rapid, the skin is moist and warm, and exophthalmos is apparent. This condition is considered to be (A) autoimmune. (B) congenital. (C) iatrogenic. (D) infectious. (E) nutritional.

The answer is A. Isolated IgA deficiency is most often asymptomatic but can be characterized by anaphylactic reactions to transfused blood. It is critical to notify the transfusion service about a patient with possible IgA deficiency prior to giving blood products so that products can be washed to remove any immunoglobulins prior to administration. This disorder can also be associated with frequent episodes of diarrhea and recurrent infections, especially those involving mucosal surfaces. This inherited B-cell defect is due to inability of IgA B cells to mature into IgA-producing plasma cells. Interestingly, the defect leading to systemic anaphylaxis involves both IgA and IgE antibody formation. Patients lacking IgA can develop IgE antibodies against the IgA antibodies present in transfused blood. This sensitization can result in susceptibility to anaphylaxis on subsequent transfusion.

A 24-year-old woman who had previously been uneventfully transfused receives a blood transfusion during surgery and shortly thereafter develops itching, generalized urticaria, laryngeal edema, and dyspnea with wheezing respiration. She has a past history of recurrent upper respiratory tract infections and frequent episodes of diarrhea. Laboratory studies are most likely to reveal decreased concentrations of which of the following immunoglobulins? (A) IgA (B) IgD (C) IgE (D) IgG (E) IgM

The answer is D. The histologic hallmark of sarcoidosis is the finding of noncaseating granulomatous inflammation. Although this finding is not entirely specific, a non-necrotizing granulomatous response of the lung is rarely seen in patients with tuberculosis or deep-seated fungal infections. These infections usually have a necrotizing component.

A 26-year-old African-American woman has bilateral hilar adenopathy, and radiography reveals multiple reticular densities in both lung fields. A bronchoscopic biopsy reveals granulomatous inflammation with multiple giant cells of the Langhans type and no evidence of caseous necrosis. Which of the following is the most likely diagnosis? (A) Aspergillosis (B) Coccidioidomycosis (c) Histoplasmosis (d) Sarcoidosis (e) Tuberculosis

The answer is C. The most common effect of maternal diabetes mellitus and hyperglycemia on the child is increased birth weight. This also increases the likelihood of obstetric complications, including the need for cesarean section and increased likelihood of brachial plexus injuries. Another complication is hyaline membrane disease. Cretinism results from deficiency of thyroid hormone during fetal development and during postnatal life. Ambiguous genitalia can occur in any of the adrenogenital syndromes. Sheehan syndrome occurs in the mother and has no relationship to diabetes. Thyroglossal duct cysts do not usually result in endocrine complications.

A 26-year-old primigravida develops gestational diabetes and remains hyperglycemic during the remainder of her pregnancy. Which of the following abnormalities in the newborn child is likely related to the maternal hyperglycemia? (A) Ambiguous genitalia (B) Cretinism (C) Increased birth weight (D) Sheehan syndrome (E) Thyroglossal duct cyst

The answer is E. The history is typical of amniotic fluid embolism, one of the major obstetric causes of disseminated intravascular coagulation (DIC). Other obstetric complications associated with DIC include retained dead fetus and abruptio placentae (premature separation of the placenta). Nonobstetric causes include neoplasms or tissue damage from infection, immunologic mechanisms, or trauma. Neoplastic causes include tumors of the lung, pancreas, prostate, and stomach, and FAB M3 acute myeloblastic (promyelocytic hypergranular) leukemia. Tissue damage can result from trauma such as lung surgery, from hemolysis or hemolytic transfusion reactions, and from inflammatory causes, such as gram-negative sepsis and immune complex disease.

A 26-year-old woman dies after a short illness beginning in the late stages of labor. At autopsy, blood vessels in the lungs contained fetal debris (e.g., squamous cells, vernix, mucin), as did other vessels of multiple organs. Review of the clinical history reveals that she had become acutely ill with dyspnea, hypotension, and seizures, and a chest radiograph had demonstrated evidence of pulmonary edema. This was all followed by prolonged hemorrhage from the vagina and generalized bleeding from multiple other sites. The changes that were found within multiple blood vessels most likely are (A) bone marrow emboli. (B) fat emboli. (C) gas emboli. (D) septic emboli. (e) widespread thrombosis.

The answer is E. Urinary vanillylmandelic acid, a norepinephrine metabolite, is markedly elevated in pheochromocytoma. Serum C-peptide is elevated in insulinoma. Serum calcitonin is sometimes used to screen for medullary carcinoma of the thyroid. Serum hemoglobin A1c is an indicator of long-term blood glucose control in diabetes mellitus. Urine aldosterone is elevated in aldosteronism, both primary and secondary.

A 26-year-old woman has episodic hypertension with headache, diaphoresis, and palpitation. Which of the following diagnostic procedures would be most useful in evaluating the possibility that a pheochromocytoma might be the cause of these findings? (A) Serum C-peptide (B) Serum calcitonin (C) Serum hemoglobin A1c (glycosylated hemoglobin) (D) Urinary aldosterone (E) Urinary vanillylmandelic acid

The answer is D. Recurrent intractable peptic ulcer disease is characteristic of the Zollinger-Ellison syndrome with excess gastrin production, most often from a gastrinoma. Cushing syndrome causes an excess of cortisol. Glucagonoma is a rare neuroendocrine tumor of pancreatic alpha cells that can cause hyperglycemia. The Whipple triad (episodic hyperinsulinemia and hypoglycemia causing CNS dysfunction reversible by glucose administration) is seen with insulinoma. Acromegaly results from growth hormone excess.

A 28-year-old man is evaluated for recurrent peptic ulcer disease, apparently refractory to pharmacologic intervention. Serum gastrin is markedly elevated. These findings are most characteristic of which of the following? (A) Cushing syndrome (B) Glucagonoma (C) Whipple triad (D) Zollinger-Ellison syndrome (E) Acromegaly

The answer is A. The combination of a prolonged activated partial thromboplastin time (APTT), a positive VDRL test for syphilis, recurrent thromboses (arterial or venous), and spontaneous abortion is highly suggestive of the antiphospholipid antibody syndrome. As the name implies, antibodies directed at phospholipids are a characteristic finding. Because of the prolonged APTT and frequent association with systemic lupus erythematosus (SLE), the antibody has been referred to as the "lupus anticoagulant," a misleading term because affected subjects have a thrombotic rather than hemorrhagic diathesis and not all subjects have SLE. The term primary antiphospholipid antibody syndrome is used when there is no evident underlying disease. It should be contrasted to secondary antiphospholipid antibody syndrome, in which the patient has a well-defined autoimmune disorder such as SLE.

A 28-year-old woman is evaluated for possible thrombophilia since she has had two episodes of deep vein thrombosis, as well as two pregnancies that terminated in spontaneous abortion. The activated partial thromboplastin time is prolonged, and she has a positive VDRL screening test for syphilis. This combination of findings is most suggestive of (A) antiphospholipid antibody syndrome. (B) disseminated intravascular coagulation. (C) factor V Leiden. (D) methylene tetrahydrofolate reductase mutation. (e) prothrombin 20210A transition.

The answer is C. T helper (TH1) cells secrete IL-2 and interferon-γ, which in turn facilitate T-cell expansion and macrophage activation.

A 28-year-old woman is found to have pulmonary sarcoidosis. Flow cytometric analysis of T cells isolated from the alveoli and lung interstitium reveals the presence of large numbers of T helper (TH1) cells. These cells are known to secrete which of the following substances? (A) Complement component C5A (B) Elastase and lysyl-hydroxylase (C) Interleukin-2 (IL-2) and interferon-γ (D) IL-8 and transforming growth factor-β (E) Leukotrienes C4and C5

The answer is A. Caseous necrosis occurs as part of granulomatous inflammation, typified by the lesions of tuberculosis.

A 29-year-old man hospitalized for acquired immunodeficiency syndrome (AIDS) is found to have pulmonary tuberculosis. Which type of necrosis is found in the granulomatous lesions (clusters of modified macrophages) characteristic of this increasingly frequent complication of AIDS? (A) Caseous (B) Coagulative (C) Enzymatic (D) Fibrinoid (E) Liquefactive

The answer is D. In the tetralogy of Fallot, the characteristic lesions include ventricular septal defect, overriding aorta, pulmonary valve stenosis, and right ventricular hypertrophy. The pulmonary stenosis and overriding aorta cause increased right ventricular pressure and lead to right-to-left shunting. Cyanosis, which occurs when the arterial concentration of reduced hemoglobin exceeds 5 mg/mL, is seen with a right-to-left shunt, in which venous blood gains direct access to the arterial circulation. In contrast, patent ductus arteriosus, atrial septal defect, and ventricular septal defect are associated with left-to-right blood flow.

A 3-year-old boy presents with cyanosis and shortness of breath that develops when he plays with friends. According to his mother, the boy was born cyanotic. The boy is very small and short for his age, and he squats on the floor next to his mother. Chest radiography reveals a boot-shaped heart, normal heart size, and a right aortic arch. Echocardiography reveals a large ventricular septal defect with an overriding aorta, pulmonary stenosis, and right ventricular hypertrophy. Which of the following is the most likely diagnosis? (A) Coarctation of the aorta (B) Patent ductus arteriosus (C) Rheumatic heart disease (D) Tetralogy of Fallot (E) Transposition of the great vessels

The answer is A. The findings described are associated with prolactinoma, the most common hormone-secreting tumor of the pituitary. Hyperprolactinemia results in amenorrhea and galactorrhea. The tumor is generally small; however, about 10% are large enough to impinge on adjacent structures, and bitemporal hemianopsia from pressure on the optic chiasm is common. Somatotropic adenoma (an older name is acidophilic adenoma) secretes growth hormone, causing acromegaly or gigantism, whereas corticotropic adenoma secretes ACTH, causing Cushing syndrome of pituitary origin (Cushing disease). Craniopharyngioma is a nonendocrine tumor of Rathke pouch origin that can be locally destructive, indirectly causing panhypopituitarism.

A 32-year-old woman presents with amenorrhea, galactorrhea, and visual field defects, all of several months' duration. Magnetic resonance imaging reveals a hypophyseal mass impinging on the optic chiasm. This is most likely a(n) (A) prolactinoma. (B) somatotropic adenoma. (C) corticotropic adenoma. (D) craniopharyngioma. (E) acidophilic adenoma.

The answer is E. The combination of hypertension, persistent hypokalemia, and slightly elevated serum sodium is highly suggestive of Conn syndrome (primary aldosteronism, hyperaldosteronism). The diagnosis can be confirmed by demonstration of increased aldosterone, lack of response of aldosterone to sodium loading, and decreased serum renin.

A 34-year-old man is referred for evaluation of hypertension and persistent hypokalemia in spite of taking oral potassium supplements. Blood pressure is 180/110 mm Hg. Serum sodium is 149 mEq/L (normal 140 to 148 mEq/L); potassium, 3.3 mEq/L (normal 3.6 to 5.2 mEq/L); bicarbonate, 29 mEq/L (normal 22 to 29 mEq/L); chloride, 103 mEq/L (normal 98 to 107 mEq/L); and urea nitrogen, 23 mg/dL (normal 7 to 18 mg/dL). Computed tomography demonstrates a 3-cm mass in the right adrenal gland. The most likely diagnosis is (A) Addison disease. (B) Cushing syndrome. (C) Sipple syndrome. (D) DiGeorge syndrome. (E) Conn syndrome.

The answer is C. The illustration demonstrates a well-circumscribed adrenal cortical adenoma. Cushing syndrome is a manifestation of hyperproduction of adrenal glucocorticoids, and when of adrenal origin, it is most often caused by adrenal cortical adenoma. Pituitary and hypothalamic causes of Cushing syndrome result in bilateral adrenal cortical hyperplasia. In contrast, Cushing syndrome caused by exogenous steroid medication results in adrenal atrophy.

A 34-year-old woman is seen because of unexplained weight gain, selectively over the trunk, upper back, and back of the neck; irregular menstrual periods; and increasing obesity. She is especially concerned about the changing contour of her face, which has become rounder, creating a "moon-faced" appearance. She has also developed purple-colored streaking resembling stretch marks over the abdomen and flanks, as well as increased hair growth in a male distribution pattern. Blood pressure is elevated to 190/100 mm Hg. Blood sugar is elevated. Computed tomography reveals a smooth, homogeneous lesion in the left adrenal gland. Surgery is performed, and the resected adrenal resembled that shown in the figure. The clinical findings and the change in the adrenal gland are most likely related to which of the following? (A) Adrenal (glucocorticoid) steroid therapy (B) Ectopic production of adrenocorticotropin (C) Hyperproduction of adrenal glucocorticoids (D) Hyperproduction of hypothalamic corticotropin-releasing factor (E) Hyperproduction of pituitary corticotropin

The answer is A. The history is strongly suggestive of panhypopituitarism due to ischemic necrosis of the pituitary, occurring as a sequela to childbirth complicated by hemorrhagic shock (Sheehan syndrome). This syndrome is clinically dominated by overt evidence of gonadotropin and corticotropin deficiencies, along with laboratory evidence of these deficiencies and thyrotropin deficiency. Overt secondary hypothyroidism sometimes occurs.

A 35-year-old woman is seen 6 months after giving birth to a normal infant. She suffered severe cervical lacerations during delivery, resulting in hemorrhagic shock. Following blood transfusion and surgical repair, postpartum recovery has so far been uneventful. She now complains of continued amenorrhea and loss of weight and muscle strength. Further investigation might be expected to demonstrate which of the following findings? (A) Decreased serum cortisol (B) Hyperestrinism (C) Hyperglycemia (D) Increased hair growth in a male distribution pattern (E) Increased serum free thyroxine

The answer is E. Graves disease is characteristically associated with decreased TSH activity. Thyroid-follicle TSH receptors are stimulated by TSI, an IgG autoantibody, not by TSH. Laboratory abnormalities in hyperthyroidism include increases in serum T4, serum T3, T3 resin uptake, and radioactive iodine uptake. Total T4 and T3 resin uptake are falling into disuse. The best screening tests for Graves disease are free T4 (elevated in Graves disease) and TSH (greatly decreased in Graves disease).

A 35-year-old woman presents with amenorrhea and weight loss despite increased appetite. The history and physical examination reveal exophthalmos, fine resting tremor, tachycardia, and warm, moist skin. Laboratory tests for thyroid function would be expected to yield a decreased value for which of the following? (A) Free T4 (B) Radioactive iodine uptake (C) T3 resin uptake (D) T3 (E) Thyroid stimulating hormone

The answer is B. "Heart failure cells" are intra-alveolar hemosiderin-laden macrophages and are indicative of marked chronic passive congestion of the lung. Red cells leak from congested alveolar capillaries into the alveoli, where they are engulfed and degraded by macrophages.

A 36-year-old man dies during cardiac surgery. He had a history of long-standing rheumatic heart disease with mitral stenosis. At autopsy, the pathologist reports findings consistent with mitral stenosis and noted the presence of "heart failure cells." This finding results from (A) activation of the coagulation cascade. (B) chronic passive congestion of the lungs. (C) hypoxic myocardial injury. (D) myocardial hyperemia.

The answer is C. The description is that of the syndrome of inappropriate ADH secretion, which is excessive release of ADH, most commonly a manifestation of small cell bronchogenic carcinoma. There are numerous other causes, including a variety of tumors, CNS disorders, trauma, and infections. The cardinal features are marked decreases in both serum sodium and osmolality, normal urine sodium, and urine osmolality considerably exceeding that of the serum.

A 36-year-old man is brought to the emergency department by his wife because of lethargy, weakness, and confusion. Serum sodium and serum osmolality are markedly decreased. Urine osmolality is increased. These findings are most likely related to (A) adenoma of the anterior pituitary. (B) adenoma of the posterior pituitary. (C) bronchogenic carcinoma. (D) diabetes insipidus. (E) Sheehan syndrome.

The answer is D. The child exhibits signs of congenital hypothyroidism (cretinism). This disorder can result from many causes, including iodine deficiency, maldevelopment of the fetal thyroid, or transplacental transfer of antithyroid antibodies from a mother with autoimmune thyroid disease (a high possibility in this clinical case). The diagnosis of congenital hypothyroidism is confirmed in infants, as in adults, by decreased serum free T4 and increased TSH. If maternal antibody transfer is suspected, then tests for antithyroid antibody testing can be performed in both the mother and child.

A 4-month-old child is brought to the pediatrician for evaluation. The mother received no prenatal care and states that she has "a thyroid condition." The child appears markedly developmentally delayed, with coarse features, macroglossia, and an umbilical hernia. The child likely has which of the following conditions? (A) Cushing disease (B) Acromegaly (C) Diabetes insipidus (D) Cretinism (E) Thyroglossal duct cyst

The answer is D. The clinical findings described in the question are typical of right-sided heart failure, as are the illustrations, which reveal the nutmeg-like appearance of hepatic chronic passive congestion. The gross morphologic appearance is caused by congested centrilobular areas alternating with pale portal areas.

A 40-year-old woman dies after a long history of an illness characterized by dyspnea, orthopnea, hepatomegaly, distended neck veins, and peripheral edema. The cut surface of the liver as it appears at autopsy is shown in the first panel. The second panel shows the microscopic appearance of the liver. Which of the following disorders is the most likely cause of these findings? (A) Chronic alcoholism (B) Diabetes mellitus (C) Niemann-Pick disease (D) Right-sided heart failure (e) Viral hepatitis

The answer is B. Cardiomyopathies are noninflammatory myocardial disorders that are not associated with coronary artery obstruction, hypertension, valvular disease, congenital heart disease, or infectious disease. They are most often characterized by otherwise unexplained ventricular dysfunction, such as cardiac failure, ventricular enlargement, or ventricular arrhythmias.

A 42-year-old man is seen because of a long history of slowly developing congestive heart failure. His blood pressure is normal. Coronary artery angiography reveals no vascular disease. No heart murmurs are heard. The white blood cell count, differential, and erythrocyte sedimentation rate are normal. The most likely diagnosis is (A) carcinoid heart disease. (B) cardiomyopathy. (C) coarctation of the aorta. (D) constrictive pericarditis. (E) myocardial infarction.

The answer is B. In advanced form, primary (hereditary) hemochromatosis is characterized by the triad of cirrhosis, diabetes, and hyperpigmentation, or so-called bronze diabetes. The disease is most often caused by a mutation in the Hfegene on chromosome 6 and is characteristically familial rather than sporadic. The manifestations of the disorder are the result of iron overload and deposition of hemosiderin in tissues such as the liver, pancreas, skin, joints, and pituitary. Laboratory abnormalities of note include increased serum iron and decreased TIBC. The skin hyperpigmentation is due largely to increases in melanin and to lesser accumulations of hemosiderin.

A 45-year-old man is referred because of a recent diagnosis of hereditary hemochromatosis. Which of the following is a correct statement about this disorder? (A) Damage to organs results from abnormal deposition of lead (B) It can progress to liver cirrhosis, diabetes mellitus, and skin pigmentation (C) Most cases are due to spontaneous mutations (D) Skin hyperpigmentation is due to bilirubin accumulation (E) The TIBC is characteristically increased

The answer is C. Pancreatic enzymatic fat necrosis represents autodigestion by proteolytic and lipolytic enzymes released from damaged parenchymal cells of the pancreas. Fatty acids liberated by the digestion of fat form calcium soaps, a process referred to as saponification. The precipitated calcium in the soaps can be visualized by radiologic imaging.

A 45-year-old man with a long history of alcoholism presents with severe epigastric pain, nausea, vomiting, fever, and an increase in serum amylase. During a previous hospitalization for a similar episode, computed tomography scanning demonstrated calcifications in the pancreas. A diagnosis of acute pancreatitis superimposed on chronic pancreatitis was made. In this condition, which of the following types of necrosis is most characteristic? (A) Caseous (B) Coagulative (C) Enzymatic (D) Fibrinoid (E) Liquefactive

The answer is B. The decreased size is due to restriction of the blood supply, one of the causes of atrophy. The increase in size of the opposite kidney is referred to as compensatory hypertrophy. Unilateral renal artery stenosis is a well-known cause of secondary hypertension. In this setting, increased renin excretion and stimulation of the renin-angiotensin system results in a form of hypertension that is potentially curable by surgical correction of the underlying vascular abnormality.

A 45-year-old woman is investigated for hypertension and is found to have enlargement of the left kidney. The right kidney is smaller than normal. Contrast studies reveal stenosis of the right renal artery. The size change in the right kidney is an example of which of the following adaptive changes? (A) Aplasia (B) Atrophy (C) Hyperplasia (D) Hypertrophy (E) Metaplasia

The answer is D. This is a classic case of stable angina, which is chest pain that is precipitated by exertion but relieved by rest. Stable angina is due to atherosclerosis of the coronary arteries. This patient has risk factors for ischemic heart disease (IHD) (e.g., cigarette smoking, hypertension, hyperlipidemia, diabetes, family history of IHD/coronary artery disease). Prinzmetal angina is intermittent chest pain at rest, and unstable angina is prolonged chest pain at rest.

A 55-year-old woman presents with complaints of chest pain. She states that the chest pain predictably occurs when she climbs four flights of stairs to reach her apartment or when she has been jogging for more than 10 minutes. She is particularly concerned because her mother died of a myocardial infarction at 50 years of age. Which of the following best describes this patient's state? (A) Arrhythmia (B) Myocardial infarction (C) Prinzmetal angina (D) Stable angina pectoris (E) Unstable angina pectoris

The answer is A. The patient exhibits the cardinal findings of the CREST syndrome, a less severe variant of systemic sclerosis (scleroderma) characterized by Calcinosis, Raynaud phenomenon, Esophageal dysfunction, Sclerodactyly, and Telangiectasia. Although a number of antinuclear antibodies can be found in this disorder, the most characteristic is directed at centromeric proteins (the antibody is often referred to as anticentromere).

A 45-year-old woman is seen because of varied complaints. She has been troubled by small painful lumps under the skin of her fingers, some of which have ruptured and leaked a chalky white substance. She also complains of painful episodes in her fingers and toes, which blanch and turn blue on exposure to cold. In addition, questioning reveals increasing "heartburn" and difficulty swallowing. Examination reveals thickening of the skin of the fingers and toes, resulting in a claw-like appearance. Telangiectatic clusters of vessels, appearing as small focal red lesions, are observed in the skin of the face, upper trunk, and hands, and on the mucosal surface of the lips. Antibodies to which of the following are most characteristic of the findings presented by this patient? (A) Centromeric proteins (B) Histidyl-t-RNA synthetase (Jo-1) (C) Histones (D) Mitochondria (E) Native DNA

The answer is C. The findings are those of acromegaly, which is caused by a pituitary somatotropic adenoma. Growth hormone excess causes elevation in concentration of IGF-I (somatomedin C), measurement of which is a reliable indicator of disease activity. The tumor can also produce local effects, the most common of which is bitemporal hemianopsia from pressure on the optic chiasm.

A 46-year-old man is referred to an endocrinologist because of the recent onset of diabetes mellitus. His overall appearance is striking, however, and on questioning, he describes marked changes that have been occurring slowly over many years. Comparison with old photographs reveals that he has developed generalized coarseness of his facial features, including frontal bossing, thickening of the nose, prognathism (enlargement and increased prominence of the jaw), and macroglossia (enlargement of the tongue). In addition, he has enlarged extremities, with sausage-like fingers, and he says that he is no longer able to wear his wedding ring and that his shoe size has increased. These findings are characteristic of increased activity of which of the following? (A) Corticotropin (B) Dopamine (C) Insulinlike growth factor-I (D) Prolactin (E) Thyroid stimulating hormone

The answer is A. This is a case of paradoxical embolism, which denotes the passage of an embolus of venous origin into the arterial circulation, by way of a right-to-left shunt (e.g., atrial septal defect or patent foramen ovale). Ordinarily, atrial septal defects result in a left-to-right shunt across the atrial septum, but over time may develop into a rightto-left shunt. The likelihood of right-to-left passage of an embolus is often enhanced by pulmonary hypertension, sometimes secondary to pulmonary thromboembolism.

A 50-year-old man presents with sudden weakness in his left leg. He has felt well lately and has no past medical history of coronary artery disease, hyperlipidemia, or hypertension, and no family history of myocardial infarction or stroke. Physical examination reveals motor weakness in the left leg, with no other neurologic deficits, and no cardiac murmur. Magnetic resonance imaging of the brain demonstrates a small ischemic infarct in the arterial distribution of the brain correlating with motor control of the left leg. Angiography and echocardiography reveal normal coronary arteries, normal valves with no vegetations, and a small right-to-left shunt. Which of the following is most likely associated with this scenario? (A) Atrial septal defect (B) Bacterial endocarditis (C) Nonbacterial thrombotic (marantic) endocarditis (D) Tetralogy of Fallot (E) Ventricular septal defect

The answer is C. The findings are consistent with occlusion of the middle cerebral artery, the most common site of arrest of arterial emboli in branches of the carotid artery. Such emboli usually arise from a mural thrombus in the left atrium or left ventricle. Left atrial mural thrombi are especially associated with mitral stenosis with atrial fibrillation. Mural thrombi in the left ventricle are caused by myocardial infarction. Thrombi at the junction of the internal and external carotid arteries are a cause of thrombotic brain infarcts and can also be a site of origin of emboli.

A 50-year-old right-handed man with a long history of rheumatic heart disease with mitral stenosis and atrial fibrillation is brought to the emergency department after collapsing to the floor at home. He is unable to speak or walk and has right hemiplegia with a right extensor plantar response. These findings most likely result from embolism to which of the following arteries? (A) Anterior cerebral (B) Anterior communicating (C) Middle cerebral (D) Posterior communicating (e) Superior cerebellar

The answer is A. The clinical findings are those of type I Gaucher disease, which is a manifestation of glucocerebrosidase deficiency. The disorder is most often seen in persons of European (Ashkenazic) Jewish lineage. Prominent findings include bone pain and fractures, easy bruising, hepatosplenomegaly, anemia, and thrombocytopenia. Bone marrow aspiration reveals numerous typical Gaucher cells, but specific enzyme assay is required to confirm the diagnosis. This lysosomal storage disease is relatively mild compared to a number of other such entities, such as Tay-Sachs disease and Niemann-Pick disease, which are rarely seen in adults. The disease is highly variable in its clinical manifestations, and assays of chitotriosidase and angiotensin-converting enzyme, markers of macrophage proliferation, are useful measures of the extent of disease and of its control.

A 50-year-old woman of Eastern European Jewish ancestry has a history of recurrent fractures and easy bruising and is found to have hepatosplenomegaly and mild anemia. Serum assays reveal elevations of chitotriosidase and angiotensin-converting enzyme. Assay of cultured leukocytes most likely reveals marked deficiency of which of the following enzymes? (A) Glucocerebrosidase (B) α-1,4-Glucosidase (C) Hexosaminidase A (D) α-L-Iduronidase (E) Sphingomyelinase

The answer is A. Congestive or dilated cardiomyopathy is the most common form of cardiomyopathy. It is characterized by four-chamber hypertrophy and dilation as well as right- and left-sided severe heart failure. In some cases, congestive (dilated) cardiomyopathy may be associated with alcoholism, thiamine deficiency, or prior myocarditis.

A 53-year-old woman presents with dyspnea on exertion, orthopnea, paroxysmal nocturnal dyspnea, edema in the legs and feet, and fatigue. She has no history of angina, other signs of coronary artery disease, hypertension, or valvular disease. Echocardiography reveals cardiomegaly, with four-chamber hypertrophy and dilation. Which of the following is the most likely diagnosis? (A) Congestive or dilated cardiomyopathy (B) Hypertrophic cardiomyopathy (C) Myocarditis (D) Restrictive cardiomyopathy

The answer is B. The phenomenon of earlier and more severe manifestations of a disorder in successive generations (anticipation) is a characteristic of many trinucleotide repeat disorders, the best known examples of which are fragile X syndrome (discussed in this chapter) and Huntington disease (described in this clinical scenario and further discussed in Chapter 23). The degree of expansion is closely related to the gender of the parent with the genetic abnormality. In the fragile X syndrome, expansion occurs during oogenesis. In Huntington disease, expansion occurs during spermatogenesis. Even though trinucleotide repeats almost always involve guanine and cytosine (G and C), the third nucleotide is different in the two conditions: CGG in fragile X syndrome and CAG in Huntington disease.

A 56-year-old man dies of a 15-year progressive illness characterized by athetoid movements and deterioration leading to hypertonicity, fecal and urine incontinence, anorexia and weight loss, and eventually dementia and death. The disease is known to have an autosomal dominant mode of inheritance and to be due to an abnormality in a gene on chromosome 4 that is altered by increased numbers of intragenic trinucleotide repeats. In addition, this disorder has an earlier onset and is more debilitating in successive generations, a phenomenon that might be due to (A) a shift from trinucleotide repeats to pentanucleotide repeats. (B) an increase in the number of trinucleotide repeats in successive generations. (C) defects in membrane receptors and transport systems. (D) imprinting variability in successive generations. (E) increased medical awareness of the condition.

The answer is E. Thromboxane A2(TxA2) promotes platelet aggregation, as does ADP. Aspirin irreversibly inhibits the enzymes cyclooxygenase 1 and 2 and thereby the synthesis of TxA2, thus inhibiting platelet aggregation, which is thought to be an important early step in atherogenesis. A negative but apparently unimportant consequence of aspirin prophylaxis is the parallel inhibition of synthesis of the antiaggregant endothelial PGI2, also a product of the cyclooxygenase pathway.

A 56-year-old man is surgically treated by a four-vessel coronary artery bypass graft procedure and placed on prophylactic daily aspirin therapy. Aspirin has been shown to prevent recurrent myocardial infarction through its ability to inhibit the synthesis of (A) adenosine diphosphate (ADP). (B) leukotriene B4(LTB4). (C) nitric oxide (NO). (D) prostaglandin I2(PGI2). (e) thromboxane A2(TxA2).

The answer is D. If infarction is averted by immediate thrombolytic therapy, indicators of necrosis, such as karyorrhexis, pyknosis, and karyolysis, which represent irreversible changes, would not be expected. Swelling of the endoplasmic reticulum from increased cell water, one of the earliest ultrastructural changes observed in injured cells, is reversible and would be expected.

A 56-year-old man recovered from a myocardial infarction after his myocardium was entirely "saved" by immediate thrombolytic therapy. If it had been possible to examine microscopic sections of his heart during his ischemic episode, which of the following would be the most likely cellular change to be found? (A) Karyolysis (B) Karyorrhexis (C) Pyknosis (D) Swelling of the endoplasmic reticulum

The answer is D. Myxoma of the heart, although rare, is the most common primary cardiac tumor. Because of the jelly-like appearance and myxoid histology similar to that of some organized thrombi, the neoplastic nature of this lesion was debated for many years; however, it is now generally believed that myxoma is a true neoplasm. The most common location of myxoma is in the left atrium. Due to its location, complications may develop due to physical obstruction of blood flow through the mitral valve, resulting in symptoms of congestive heart failure. Note that while angiosarcoma is the most common primary cardiac malignancy, it is not the most common primary cardiac tumor.

A 56-year-old woman presents with dyspnea on exertion, orthopnea, paroxysmal nocturnal dyspnea, and pulmonary edema. She also presents with severe dizziness and syncope, fatigue, weight loss, and arthralgias. After undergoing several tests, she is diagnosed with a primary heart tumor that is causing a "ball-valve obstruction" of her mitral valve. Which of the following is the most likely tumor? (A) Fibroma (B) Leiomyoma (C) Lipoma (D) Myxoma (E) Rhabdomyoma

The answer is A. This is a case of syphilitic (luetic) aortitis. In syphilitic aortitis, the elastica of the aorta undergoes calcification and is replaced by fibrous tissue, resulting in dilation of the ascending aorta and separation of the aortic valve commissures, with resultant aortic insufficiency. Thus, echocardiography and computed tomography of the heart reveal calcification in a linear pattern along the ascending aorta, calcification in the coronary arteries (leading to anginal symptoms), and aortic valvular insufficiency.

A 60-year-old man presents with angina. He has no past medical history of heart disease. On questioning, the patient reveals that he had repeated sexually transmitted diseases in the past, including a painless chancre (a hard, round sore) on his penis, for which he never sought medical attention. Rapid plasma reagin (RPR), Venereal Disease Research Laboratory (VDRL) slide test, and fluorescent treponemal antibody (FTA) serologic tests (indicative of syphilis infection) are positive. Echocardiography and computed tomography of the heart are performed. The history of untreated syphilis suggests that these tests will most likely detect which of the following abnormalities? (A) Aortic valvular insufficiency and linear calcification along the ascending aorta (B) Bicuspid aortic valve with aortic stenosis (C) Large valvular vegetations from bacterial endocarditis (D) Right-sided heart failure from the carcinoid syndrome (E) Small fibrin deposits on the mitral valve from nonbacterial thrombotic (marantic) endocarditis

The answer is D. Glycoprotein IIb-IIIa inhibitors prevent the action of the corresponding platelet surface receptor glycoprotein complex, which is required for formation of fibrinogen bridges between adjacent platelets.

A 60-year-old man with unstable angina (a form of acute coronary syndrome) is treated with an intravenously administered glycoprotein IIb-IIIa inhibitor. The mechanism of action of this agent is the ability to (A) dilate coronary arteries. (B) inhibit atherogenesis. (C) inhibit platelet adhesion. (D) inhibit platelet aggregation. (e) lyse thrombi.

The answer is D. Metastatic calcification, or deposition of calcium in previously normal tissue, is caused by hypercalcemia. In this patient, tumor metastases to the bone with increased osteolytic activity caused mobilization of calcium and phosphate, resulting in hypercalcemia. Metastatic calcification should be contrasted with dystrophic calcification, in which the serum calcium concentration is normal and previously damaged tissues are the sites of deposition.

A 60-year-old woman with breast cancer and widespread bony metastases is found to have calcification of multiple organs. The calcifications are best described as (A) dystrophic with decreased serum calcium. (B) dystrophic with increased serum calcium. (C) metastatic with decreased serum calcium. (D) metastatic with increased serum calcium.

The answer is D. Rupture of the left ventricle, a catastrophic complication of acute myocardial infarction, usually occurs when the necrotic area has the least tensile strength, about 4 to 7 days after an infarction, when repair is just beginning. The anterior wall of the heart is the most frequent site of rupture, usually leading to fatal cardiac tamponade. Internal rupture of the interventricular septum or of a papillary muscle may also occur. The risk of arrhythmia is greatest within the first 6 hours after myocardial infarct. Arrhythmias are the most important early complication of acute myocardial infarction, accounting for almost 50% of deaths shortly after myocardial infarction. Myocardial, or pump, failure and mural thrombosis are other complications that may develop as a result of permanent damage to the heart after infarct. Ventricular aneurysms may develop in the fibrotic scar within 3-6 months after myocardial infarct.

A 60-year-old-man is discharged after being observed in the hospital for 4 days following a myocardial infarction. He returns to his normal activities, which include sedentary work only. This point in time following a myocardial infarct is noteworthy for the special danger of which of the following? (A) Arrhythmia (B) Mural thrombosis (C) Myocardial (pump) failure (D) Myocardial rupture (E) Ventricular aneurysm

The answer is B. The term cor pulmonalerefers to right ventricular hypertrophy caused by pulmonary hypertension secondary to disorders of the lungs or pulmonary vessels. Other causes of right ventricular hypertrophy and failure, such as valvular disease, congenital defects, and left-sided heart failure, are precluded by this definition. Therefore, although in general, the most common cause of right-sided heart failure is left-sided heart failure, cor pulmonale with right-sided heart failure is due to an intrinsic disease originating in the lungs. Constrictive pericarditis can clinically mimic right-sided heart failure but is entirely unrelated to cor pulmonale.

A 64-year-old woman presents with dependent peripheral edema in her ankles and feet. She has long-standing chronic obstructive lung disease and a long history of cigarette smoking. Further investigation reveals that she has cor pulmonale with right-sided heart failure. Which of the following is the most likely cause of the right-sided heart failure in this patient? (A) Constrictive pericarditis (B) Disease of the lungs or pulmonary vessels (C) Left-sided heart failure (D) Pulmonary infundibular or valvular stenosis (E) Systemic hypertension

The answer is E. Liquefactive necrosis is characteristic of ischemic injury in the CNS and suppurative infections that cause abscess formation (see Chapter 2). The changes in the cerebrospinal spinal fluid characteristic of bacterial meningitis are detailed in Chapter 3.

A 64-year-old woman presents with fever, chills, headache, neck stiffness, vomiting, and confusion. The Kernig sign (passive knee extension eliciting neck pain) and Brudzinski sign (passive neck flexion eliciting bilateral hip flexion) are both positive. Examination of the cerebrospinal fluid reveals changes consistent with bacterial meningitis, and brain imaging demonstrates a localized abscess. Which of the following types of necrosis is most characteristic of abscess formation? (A) Caseous (B) Coagulative (C) Enzymatic (D) Fibrinoid (E) Liquefactive

The answer is E. Severe combined immunodeficiency disease is characterized by failure to thrive and increased susceptibility to bacterial, fungal, and viral infections. Laboratory studies reveal decreased numbers of both B cells and T cells and deficiency of immunoglobulins. The treatment of choice is bone marrow (or other sources of hematopoietic stem cells) transplantation and is based on maturation of donor lymphoid progenitor cells.

A 7-month-old boy has had multiple bouts of otitis media, sinusitis, bronchitis, oral candidiasis, and multiple viral infections. Cessation of the recurrent infections follows successful engraftment of a bone marrow transplant. The basis of the clinical improvement is (A) direct transfusion of antibodyproducing B cells. (B) direct transfusion of donor CD4+ and CD8+ lymphocytes. (C) donor suppression of recipient cytotoxic T cells. (D) infusion of donor-derived cytokines. (E) maturation of donor lymphoid progenitor cells.

The answer is C. The clinical findings are those of "stroke," or cerebrovascular disease. This group of entities encompasses injury to the brain caused by disorders of the cerebral vasculature, such as thrombosis, embolism, and hemorrhage (see Chapter 3). The most important consequence is damage to neurons, because neurons are considered to be "permanent" cells, incapable of division and replication (however, this has been recently challenged as the result of provocative stem cell research). Permanent cells are exemplified by neurons and myocardial cells. Labile cells, such as cells of the epidermis and gastrointestinal mucosa, divide throughout the life of the individual. Stable cells, such as hepatocytes and renal tubular cells, do not divide regularly but have the capacity to divide and regenerate as needed.

A 70-year-old man presents with the sudden onset of left-sided weakness, spasticity, and hyperactive and pathologic reflexes. The most serious consequences of this disorder are the result of damage to which of the following cell types? (A) Labile cells (B) Multipotent adult progenitor cells (c) Permanent cells (d) Stable cells

The answer is B. A clear, straw-colored fluid with low protein and low specific gravity is a transudate, and the term hydrothorax refers to the accumulation of a significant volume of transudate within the pleural cavities (to be detected by chest radiograph, about 200 to 400 mL of pleural fluid must be present). The most common cause of hydrothorax is cardiac failure, which may be either unilateral or bilateral (bilateral is more common). It is incumbent on the clinician to distinguish pleural transudates from exudates, because the causes of each are quite different.

A 70-year-old man seeks medical attention because of shortness of breath on minimal exertion. A posteroanterior chest radiograph reveals blunting of the right costophrenic sulcus interpreted as a rightsided pleural effusion. The aspirated fluid is straw colored and clear. The protein concentration is low, and the specific gravity is 1.011. Microscopic examination reveals an occasional mesothelial cell. Which of the following is the most likely cause of the effusion? (A) Decreased oncotic pressure (B) Left ventricular heart failure (C) Mesothelioma (D) Pneumonia (e) Tuberculosis

The answer is D. Nonbacterial thrombotic endocarditis, or marantic endocarditis, has been associated with a variety of wasting diseases and is observed most often in patients with cancer.

A 70-year-old woman has a long history of metastatic colon cancer, and she donates her body for use in medical school anatomy courses. At death, the body is emaciated and cachectic, and gross dissection reveals small fibrin deposits arranged around the line of closure of the leaflets of the mitral valve. The valvular lesions most likely represent (A) bacterial endocarditis. (B) endocarditis of the carcinoid syndrome. (C) Libman-Sacks endocarditis. (D) nonbacterial thrombotic (marantic) endocarditis. (E) rheumatic endocarditis.

The answer is E. The patient has bacterial pneumonia due to Streptococcus pneumoniae, a classic example of severe acute inflammation. In the early stages of acute inflammation, the neutrophil is the most prominent inflammatory cell. It is noteworthy that, in many instances, bacterial infections are characterized by neutrophilic infiltrates. It is also noteworthy that S. pneumoniae(also known as the "pneumococcus") is the most common etiologic agent of lobar pneumonia (see Chapter 14).

A 72-year-old man presents with a 3-day history of progressively worsening productive cough, fever, chills, and signs of toxicity. Prominent physical findings include signs of consolidation and rales over the right lung base. Sputum culture is positive for Streptococcus pneumoniae.An intra-alveolar exudate filling the alveoli of the involved portion of the lung is present. Which of the following types of inflammatory cells is most likely a prominent feature of this exudate? (A) Basophils (B) Eosinophils (c) Lymphocytes (d) Monocytes-macrophages (e) Neutrophils

The answer is C. The most common cause of death that occurs during acute rheumatic fever is cardiac failure secondary to myocarditis.

A 9-year-old girl is diagnosed with acute rheumatic fever. Instead of recovering as expected, her condition worsens, and she dies. Which of the following is the most likely cause of death? (A) Central nervous system involvement (B) Endocarditis (C) Myocarditis (D) Pericarditis (E) Streptococcal sepsis

The answer is B. The clinical description is characteristic of pulmonary infarction, which, in turn, most often results from thromboembolism originating from thrombosis in the lower extremity veins. Because venous thrombosis is associated with impaired blood flow, this condition is particularly characteristic of immobilization, which is often seen in elderly, debilitated, or chronically bedridden persons.

A bedridden elderly patient experiences the sudden onset of pleuritic pain and hemoptysis. The underlying lesion that led to this complication was most likely located in which of the following sites? (A) Hepatic veins (B) Lower extremity veins (C) Pelvic veins (D) Portal vein (e) Pulmonary veins

The answer is A. Several substances have chemotactic potential for neutrophils (see Table 2-1). C5a is a prominent example.

A laboratory experiment is performed to evaluate the chemotactic potential of a group of potential mediators. Which of the following substances most likely has the greatest affinity for neutrophils? (A) C5a (B) Fucosyl transferase (c) β2-Integrin (d) P-selectin (e) TNF-α

The answer is C. In mitochondrial inheritance, inheritance is entirely maternal in transmission. Affected males do not transmit the trait to any of their children, and affected females transmit the trait to all of their children. Abnormalities of mitochondrial inheritance typically involve genes that code for enzymes of oxidative phosphorylation.

A newly described neurologic disorder is found to affect multiple family members in three generations that were available for study. In the first generation, two sisters and one brother were affected. In the second generation, all of the children of the first-generation sisters were affected, but none of the descendants of the first-generation son. In the third generation, all of the children of the affected second-generation women were affected, but none of the descendants of the secondgeneration men. The mode of inheritance exemplified here is (A) autosomal dominant. (B) autosomal recessive. (C) mitochondrial. (D) X-linked dominant. (E) X-linked recessive.

The answer is C. Medullary carcinoma of the thyroid is a calcitonin-producing tumor of C cells of the thyroid. Calcitonin contributes to amyloid deposition within the tumor. Tumor cells with "Orphan Annie" nuclei and the presence of psammoma bodies are seen in papillary carcinoma of the thyroid. Infiltrates of lymphocytes with germinal center formation are seen in Hashimoto thyroiditis. In Riedel thyroiditis, the thyroid is replaced by fibrous tissue and can clinically mimic carcinoma.

A palpable mass is noted in the right lobe of the thyroid of a 45-year-old man who visits his physician for a periodic checkup. A biopsy is performed and results in a diagnosis of medullary carcinoma of the thyroid. Which of the following histologic features of thyroid disease would most likely be present in this biopsy specimen? (A) Tumor cells with "Orphan Annie" nuclei (B) Psammoma bodies (C) Tumor cells embedded in an amyloid-laden stroma (D) Infiltrates of lymphocytes with germinal center formation (E) Replacement of the thyroid with fibrous tissue

The answer is B. A positive Congo red test confirms the presence of amyloid. Apple green birefringence is observed under polarized light.

A pathologist examines a renal biopsy from a 45-year-old man with nephrotic syndrome and requests a Congo red stain to confirm the nature of an amorphous acidophilic extracellular hyaline substance localized within the mesangial matrix of the glomeruli. A positive test confirms the presence of (A) α1-antitrypsin. (B) amyloid. (C) copper. (D) glycogen. (E) hemosiderin.

The answer is B. This type of reaction is primarily mediated by the release of histamine from tissue mast cells, and the associated cellular infiltrate and peripheral blood findings represent mobilization and increased numbers of eosinophils. The symptoms reported are those of seasonal rhinitis, better known as "hay fever," a manifestation of type I hypersensitivity (see Chapter 5).

A routine complete blood count performed on a 22-year-old medical student reveals an abnormality in the differential leukocyte count. She has been complaining of frequent sneezing and "watery" eyes during the past several weeks and reports that she frequently had such episodes in the spring and summer. Which of the following cell types is most likely to be increased? (A) Basophils (B) Eosinophils (c) Lymphocytes (d) Monocytes (e) Neutrophils

The answer is E. The concentration of phenylalanine in affected infants is usually normal at birth and increases rapidly during the first days of life. False-negative results are common immediately after birth but are rare on the second and third days of life. Consequently, the blood sample for phenylketonuria is usually taken from the infant's heel within 2 to 3 days after birth. If the test is performed too early, the diagnosis could be missed.

A screening test for phenylketonuria (PKU) is performed on umbilical cord blood from a fair-skinned blond, blue-eyed infant born to dark-complexioned parents. The test is reported as negative, and no dietary restrictions are imposed. At 1 year of age, the child is seen again, this time with obvious signs of severe mental retardation, and a diagnosis of PKU is made. The diagnosis was missed at birth because (A) cord blood is not a good source of fetal blood. (B) the screening (Guthrie) test has low sensitivity. (C) the test should have been performed on maternal blood. (D) the test should have been performed on urine rather than blood. (E) the test was performed too early.

The answer is C. Deficiency of 21-hydroxylase is the cause of the most common of the adrenogenital syndromes. This enzyme deficiency results in decreased cortisol, decreased mineralocorticoids, and an increase in sex hormones, with resultant salt-losing hypotension and virilization (masculinization). Deficiency of 11-hydroxylase causes clinical findings similar to those of 21-hydroxylase deficiency, except for hypertension secondary to increased deoxycorticosterone, which has aldosterone-like activity. Deficiency of 17-hydroxylase results in decreased cortisol, increased mineralocorticoids, and decreased sex hormones. Amylin accumulates in diabetic islets, and 1α-hydroxylase is deficient in vitamin D-dependent rickets (type I).

A tentatively female newborn has ambiguous genitalia. What appears to be a vagina is associated with a significantly enlarged clitoris resembling a penis. Other findings include hyponatremia, hyperkalemia, and hypotension. Deficiency of which of the following is suggested by these findings? (A) 11-Hydroxylase (B) 17-Hydroxylase (C) 21-Hydroxylase (D) Amylin (E) 1α-Hydroxylase

The answer is B. The lesion shown is a papillary carcinoma of the thyroid, which is the most common form of thyroid cancer. This tumor most often remains localized to the thyroid and adjacent tissues for many years, even when local lymph nodes are involved. Papillary carcinoma is almost always nonfunctional.

A tumor similar to that shown in the illustration is observed in a biopsy specimen from the thyroid of a 50-year-old woman. An adjacent lymph node is also involved. Which of the following descriptions of this tumor is most appropriate? (A) Functional tumor resulting in thyrotoxicosis (B) Slow-growing lesion with relatively good prognosis (C) Origin from C cells (D) Calcitonin-producing tumor (E) Tumor with amyloid-containing stroma

The answer is A. Distinguishing endogenous insulin production from exogenous insulin (therapeutically or surreptitiously administered) is done by quantitation of C-peptide, a fragment of the proinsulin molecule split off during the synthesis of insulin. Circulating C-peptide is characteristically increased in patients with insulinoma. C-peptide is not increased by exogenous insulin administration because it is removed during the purification of commercial insulin preparations.

After suffering a seizure, a 23-year old woman is found to have profound hypoglycemia. Determination of which of the following would aid in differentiating exogenous hyperinsulinemia from endogenous hyperinsulinemia? (A) C-peptide (B) Gastrin (C) Glucagon (D) Proinsulin (E) Vasoactive intestinal peptide

The answer is E. The diagnosis is septic shock, most likely a result of gram-negative sepsis originating from a urinary tract infection. Gram-negative organisms contain lipopolysaccharide in the outer membrane, which triggers the release of cytokines, such as tumor necrosis factor, resulting in a cascade of events culminating in increased capillary permeability and redistribution of circulatory volume into the interstitium. Anaphylactic shock is a result of a type I hypersensitivity. Cardiogenic shock often results from myocardial infarction. Hypovolemic shock is a result of blood or fluid loss. Neurogenic shock can result from spinal cord injuries.

An 86-year-old man with a history of recurrent urinary tract infection presents with fever, tachypnea, tachycardia, mental obtundation, and reduced blood pressure. Which of the following forms of shock is most likely? (A) Anaphylactic shock (B) Cardiogenic shock (C) Hypovolemic shock (D) Neurogenic shock (e) Septic shock

The answer is E. The tuberculin test is a classic example of delayed hypersensitivity, a form of cell-mediated hypersensitivity involving CD4+T cells and macrophages. Native CD4+T cells are converted to TH1 cells that secrete cytokines, especially interferon-γ, which is a central mediator of delayed hypersensitivity. Among the many actions of interferon-γ, the most important is the activation of macrophages.

An HIV-positive intravenous drug user is suspected of having active tuberculosis, and a tuberculin (Mantoux) intradermal skin test is performed. After 48 hours, 10 cm of induration is observed. Which of the following are involved in this form of hypersensitivity reaction? (A) B cells and antibodies (B) Basophils and IgE (C) Immune complexes and complement (D) Plasma cells and IgM (E) T cells and macrophages

The answer is D. The patient presents with signs and symptoms of the WaterhouseFriderichsen syndrome, a devastating consequence of disseminated meningococcal infection. The disease is characterized by hemorrhagic destruction of the adrenals complicated by disseminated intravascular coagulation.

An acutely ill 18-year-old female college student is brought to the emergency department by her roommate. The patient is febrile and markedly hypotensive, and her mental status is obtunded. Numerous petechial and purpuric hemorrhages are scattered over the trunk, and aspiration of a lesion reveals neutrophils engulfing gram-negative diplococci. Serum sodium is markedly decreased, and serum potassium is increased. Coagulation testing reveals increased prothrombin time, activated partial thromboplastin time, and fibrin-fibrinogen split products. Which of the following is most likely? (A) Conn syndrome (B) Hyperprolactinoma (C) Neuroblastoma (D) Waterhouse-Friderichsen syndrome (E) Sipple syndrome

The answer is C. All of the findings listed are characteristic of diabetes mellitus, but only insulitis is specific for type 1 diabetes. Amylin deposition in the pancreatic islets, derived from insulin-associated polypeptide, is found especially in type 2 diabetes mellitus. Armanni-Ebstein lesions (deposition of glycogen in renal tubules) are seen in uncontrolled hyperglycemia, which can occur in either type. Glomerular Kimmelstiel-Wilson nodules are seen in long-standing diabetes, regardless of type. Similarly, proliferative retinopathy is a complication of both forms of diabetes.

An autopsy is performed on an 8-yearold child with diabetes mellitus of recent onset who has died en route to the hospital following an automobile accident. Which of the following autopsy findings would favor the diagnosis of type 1 diabetes as contrasted to type 2 diabetes? (A) Amylin deposition in pancreatic islets (B) Armanni-Ebstein lesion (C) Insulitis (D) Kimmelstiel-Wilson nodules (E) Proliferative retinopathy

The answer is E. The sequence of events in hypoxic cell damage is as follows: Hypoxia results in failure of oxidative phosphorylation, with resultant depletion of ATP and increase in adenosine monophosphate and adenosine diphosphate. Anaerobic glycolysis and glycogenolysis are stimulated (notinhibited) through increased phosphofructokinase and phosphorylase activities, respectively. This results in an accumulation of cell lactate, with a decrease in intracellular pH and depletion of cellular glycogen stores. Decreased availability of ATP also results in failure of the Na+K+-ATPase pump, which then leads to increased cell Na+ and water and decreased cell K+.

An impending myocardial infarction was successfully averted by thrombolytic (clot-dissolving) therapy in a 55-year-old man. Which of the following biochemical events most likely occurred during the period of hypoxia? (A) Decreased hydrogen ion concentration (B) Increase in oxidative phosphorylation (C) Loss of intracellular Na+ and water (D) Stimulation of ATP synthesis (E) Stimulation of anaerobic glycolysis and glycogenolysis

The answer is C. Multifactorial disorders are among the most common familial abnormalities and are much more common than monogenic disorders. They include a number of common entities, such as ischemic heart disease, diabetes mellitus, hypertension, gout, schizophrenia, bipolar disorder, and neural tube defects.

As part of a fourth-year elective, a medical student rotating through a medical genetics service is assigned to counsel a patient who is concerned about a family history of hypertension. To be properly prepared for the counseling session, the student reviews course notes on modes of inheritance of various disorders. Knowledge of which of the following modes of inheritance is most pertinent to the upcoming discussion with the patient? (A) Autosomal dominant (B) Autosomal recessive (C) Multifactorial (D) X-linked dominant (E) X-linked recessive

The answer is A. von Willebrand factor is required for platelet adhesion to the subendothelium of damaged blood vessels. See further discussion of von Willebrand disease in Chapter 13.

During a laboratory exercise on coagulation testing, a 23-year-old medical student is found to have a prolonged bleeding time. She has had a long history of "easy bleeding," with frequent bleeding of the gums, epistaxis, cutaneous bleeding, and menorrhagia. Further testing revealed a deficiency of von Willebrand factor. Which of the following thrombogenic processes involving platelets is most directly impaired? (A) Adhesion (B) Conformational change with activation of phospholipid surface (C) Formation of fibrinogen bridges (D) Release reaction (e) Stabilization of platelet plug

The answer is A. The patient has homogentisic oxidase deficiency, a rare inborn error of metabolism (actually the first such disorder described by Garrod in 1902), clinically manifest by alkaptonuria and ochronosis. The term alkaptonuria refers to urinary excretion of unmetabolized homogentisic acid imparting a dark color to urine on standing. The term ochronosis refers to pigment deposition in multiple tissues, most prominently in cartilage and connective tissue. Most symptoms result from joint involvement, which can lead to disabling arthritis as patients age. Other affected structures include the eyes, larynx and bronchi, heart and vessels, prostate, and sweat glands.

During a routine physical examination, a 41-year-old woman is noted to have blueblack pigmented patches in the sclerae and gray-blue discoloration of the ear cartilages. The extensor tendons of the hands exhibit similar discoloration when she is asked to "make a fist." On questioning, the patient vaguely remembers hearing her mother say that the patient had dark discoloration on her diapers when she was an infant. Her only current complaint is slowly increasing pain and stiffness of the lower back, hips, and knees. A urine sample darkens on standing. These findings are characteristic of a deficiency of which of the following enzymes? (A) Homogentisic oxidase (B) Hypoxanthine-guanine phosphoribosyltransferase (C) L-Iduronosulfate sulfatase (D) Ketoacid decarboxylase (E) Phenylalanine hydroxylase

The answer is D. The history is strongly suggestive of idiopathic myxedema. Expected laboratory abnormalities include decreased serum free T4, increased TSH, and increased cholesterol. Also, because hypothyroidism, with secretion of less thyroid hormone, results in less saturation of binding sites on TBG (or increased unbound binding sites), the T3 resin uptake, which is inversely proportional to the number of unbound sites, will be decreased. Note:Total T4 and T3 resin uptake, although falling into clinical disuse, may still appear in examination questions. The most appropriate test today would be TSH (which is greatly elevated in hypothyroidism) and free T4 (which is greatly reduced in hypothyroidism).

During a yearlong training program, a 23-year-old female Air Force officer falls in class rank from first place to last place. She has also noted a lower pitch to her voice and coarsening of her hair, along with an increased tendency toward weight gain, menorrhagia, and increasing intolerance to cold. Which of the following laboratory abnormalities is expected? (A) Increased serum free T4 (B) Increased serum T3 resin uptake (C) Increased saturation of thyroid hormone-binding sites on TBG (D) Increased serum TSH (E) Decreased serum cholesterol

The answer is B. The patient has cirrhosis of the liver secondary to chronic alcoholism. A prominent manifestation of this disorder is decreased hepatic synthesis of albumin, the most significant contributor to plasma oncotic pressure. In addition, ascites is associated with increased sodium and water retention because of stimulation of the renin-angiotensin system. Also, hydrostatic forces (because of intrahepatic scarring and partial obstruction of the portal venous return) result in fluid transudation and increased secretion of hepatic lymph.

Fluid is aspirated from the grossly distended abdomen of a 47-year-old chronic alcoholic man. The fluid is straw colored and clear and is found to have a protein content (largely albumin) of 2.5 g/dL. Which of the following is a major contributor to the fluid accumulation in this patient? (A) Blockage of lymphatics (B) Decreased oncotic pressure (C) Decreased sodium retention (D) Increased capillary permeability (e) Inflammatory exudation

The answer is D. PGI2 is a prostaglandin that is synthesized and expressed primarily in endothelial cells. It is a product of the cyclooxygenase pathway of arachidonic acid metabolism, which is inhibited by aspirin. PGI2 is a potent vasodilator and platelet antiaggregant. These properties are often contrasted with those of TxA2, which is primarily synthesized in platelets and is a vasoconstrictor and platelet aggregant. The other compounds are products of the lipoxygenase pathway of arachidonic acid metabolism, which is not inhibited by aspirin.

In a laboratory exercise for medical students, an unknown compound is studied. The students are informed that the compound has been isolated from endothelial cells and that its synthesis can be inhibited by aspirin. In the laboratory, the students demonstrate that the compound is a potent vasodilator and platelet antiaggregant. Given these findings, the substance is most likely which of the following mediators? (A) 5-HPETE (B) LTC4 (c) LXA4 (d) PGI2 (e) TxA2

The answer is C. The clinical description and the figure are both typical of advanced secondary tuberculosis. Although this disorder is now relatively uncommon, its incidence is increasing, especially in association with immunodeficiency. Tuberculosis is a classic cause of granulomatous inflammation, which is characterized by the presence of "granulomas," which by definition consist of clusters of modified macrophages referred to as epithelioid cells. Additional features such as caseous necrosis, giant cell formation, and identifiable etiologic agents may or may not be present and are not invariable features of this form of inflammation. Granulation tissue is a feature of early repair and is totally unrelated to granulomatous inflammation.

The accompanying figure is representative of the findings in a hilar lymph node from a 54-year-old man who sought medical care for low-grade fever, anorexia, fatigue, night sweats, weight loss, and persistent cough with bouts of hemoptysis. A chest x-ray had revealed a right apical infiltrate with beginning cavitation, and examination of the sputum had revealed acid-fast bacilli. This condition is typified by a form of inflammation that invariablyincludes which of the following? (A) A morphologically identifiable etiologic agent (B) Caseous necrosis (c) Clusters of epithelioid cells (d) Multinucleated giant cells (e) Prominent granulation tissue

The answer is A. The figure illustrates fatty change of the liver, which is characterized by the accumulation of intracellular parenchymal triglycerides. It is seen most frequently in the liver, heart, and kidney and is commonly secondary to alcoholism. Fatty change results from an imbalance between the uptake, utilization, and mobilization of fat from liver cells. Alcoholic fatty liver may be reversible with complete abstinence from alcohol.

The illustration is from a liver biopsy of a 34-year-old woman with a long history of alcoholism. Which of the following is the best explanation for the changes shown here? (A) Accumulation of triglycerides within hepatocytes (B) Apoptosis with replacement of damaged cells by lipid-laden macrophages (C) Bilirubin accumulation with mobilization of fat by bile salts (D) Enzymatic fat necrosis with digestion of liver parenchyma by released enzymes (E) Irreversible damage to mitochondria

The answer is D. The illustration shows marked hypertrophy of the left ventricle. Hypertrophy of this extent, often seen in hypertensive heart disease, is caused by increased workload from increased ventricular pressure. This organ enlargement is the result of an increase in size of the individual muscle cells.

The illustration shows a section of the heart from a 45-year-old African-American man with long-standing hypertension who died of a "stroke." Which of the following adaptive changes is exemplified in the illustration? (A) Aplasia (B) Atrophy (C) Hyperplasia (D) Hypertrophy (E) Hypoplasia

The answer is A. The figure illustrates an Aschoff body, the characteristic lesion of rheumatic fever. This myocardial lesion is most often oval in shape and characterized by swollen, fragmented collagen and fibrinoid material and by characteristic large mesenchymal cells (Anitschkow myocytes) and multinucleated cells (Aschoff cells). Sydenham chorea is a major manifestation of rheumatic fever.

The myocardial lesions shown in the figure were observed at the autopsy examination of a pediatric patient who died after a short illness. During life, which of the following manifestations of his illness was most likely? (A) Chorea (B) Systemic embolization (C) Systemic lupus erythematosus (D) Unstable angina (E) Wasting diseases

The answer is C. Patients with Down syndrome are at increased risk of lymphoblastic leukemia. In addition, there is common occurrence of congenital heart disease, especially defects of the endocardial cushion (atrioventricular valve malformations and atrial and ventricular septal defects), and increased susceptibility to infection. Many patients with Down syndrome who are older than 35 years of age show clinical signs, symptoms, and pathologic findings of Alzheimer-type dementia, with an incidence much higher than in the general population.

The parents of a 17-year-old boy with Down syndrome seek counseling because they are concerned that their son may develop a life-threatening disorder known to be associated with his chromosomal abnormality. The physician should be prepared to discuss which of the following disorders in terms of its association with Down syndrome? (A) Berry aneurysm of the circle of Willis (B) Creutzfeldt-Jakob disease (C) Lymphoblastic leukemia (D) Medullary carcinoma of the thyroid (E) Osteosarcoma

The answer is B. The figure illustrates general preservation of myocardial architecture with some fragmentation, more intense cytoplasmic staining corresponding to increased cellular eosinophilia, and loss of nuclei, all of which are characteristics of coagulative necrosis.

This figure illustrates the microscopic appearance of the heart of a 56-year-old man who died after a 24-hour hospitalization for severe "crushing" chest pain complicated by hypotension and pulmonary edema. The type of necrosis shown is best described as (A) caseous. (B) coagulative. (C) fibrinoid. (D) gangrenous. (E) liquefactive.

This is the answer side of the first card in this set. Make sure you select "definition" in the "start with" box. Otherwise you will just see the answers first, which isn't super productive. (I had to do this because quizlet only allows images on the back of cards. Which is dumb. But we work with what we've got).

This is the question side of the first card in this set. Make sure you select "definition" in the "start with" box. Otherwise you will just see the answers first, which isn't super productive. (I had to do this because quizlet only allows images on the back of cards. Which is dumb. But we work with what we've got). I also recommend "flow" instead of "flip," but that's just a personal preference.

The answer is D. The clinical description is characteristic of systemic anaphylaxis, an IgE-mediated type I hypersensitivity reaction. In type I hypersensitivity, reaction of antigen with preformed IgE antibodies fixed by Fc receptors to the surface of basophils or tissue mast cells results in cytolysis and degranulation of these cells, with release of histamine and other mediators.

Within minutes of a bee sting, a 23-year-old woman develops generalized pruritus and hyperemia of the skin, followed shortly by swelling of the face and eyelids, dyspnea, and laryngeal edema. This reaction is mediated by (A) antigen-antibody complexes. (B) cytotoxic T cells. (C) IgA antibodies. (D) IgE antibodies. (E) IgG antibodies.

The answer is A. By 24 hours well-developed microscopic changes of coagulative necrosis can be detected in infarcted tissue. There is loss of nuclei in cells and infiltration of neutrophils into tissue.

Yesterday, a 60-year-old man presented to the emergency department with dyspnea, diaphoresis, and crushing substernal chest pain that radiated to his neck and left arm. When asked to describe the pain, he put his fist to the center of his chest and stated that it felt "as if someone is squeezing my heart." An electrocardiogram demonstrated changes consistent with myocardial infarction, and serum troponin I levels were elevated. If the patient unexpectedly dies today, which of the following would almost certainly be found on histologic examination of the affected myocardium? (A) Coagulative necrosis with neutrophil infiltration (B) Fibrotic tissue replacing infarcted tissue (C) No histologic changes (D) Slight swelling of tissue and change of color (E) Young fibroblasts and new vessels growing into the infarcted tissue


Related study sets

Climate and Weather - Climate Change Final Unit 1

View Set

Latent Print Examiner Skills Test

View Set

Nike US data engr via GenPact 20231018 1500

View Set

IV Safety Alerts & IV Therapy Questions

View Set

Chapter 2: Chemical basis of Life: Atoms, Molecules, & water

View Set

PrepU Fluid and Electrolytes: Balance and Disturbance

View Set